Creando un estado QM de posición definida en el espacio Fock

Me pregunto si alguien podría ayudarme a terminar un cálculo simple. Permítanme primero brindar motivación para la siguiente pregunta: me gustaría escribir una amplitud QM en el 'estilo QFT', como

ϕ 2 ( t 2 ) ϕ 1 ( t 1 ) = 0 | ϕ ^ 2 ( 0 ) mi i H ( t 2 t 1 ) ϕ ^ 1 | 0
Aquí supuse que ϕ ^ 1 , 2 son hermíteos, lo que en general no tiene por qué ser necesariamente cierto.

Bien, ahora la pregunta: me gustaría saber qué operador crea los estados propios de posición en QM. Aquí está mi intento de encontrar sus elementos de matriz en la representación de posición:

O ^ | 0 = | X (por definición) X | O ^ | 0 = X | X = d ( X X ) d X X | O ^ | X X | 0 = d ( X X )

Aquí viene la dependencia del hamiltoniano: no podemos continuar sin especificar la forma particular de H , y así de X | 0 . Por supuesto, por simplicidad comenzamos con el oscilador armónico.

X | 0 = π 1 / 4 mi X 2 / 2

donde elegí metro ω = 1 por conveniencia. Por lo tanto, se obtiene:

π 1 / 4 d X X | O ^ | X mi ( X ) 2 / 2 = d ( X X )

¿Alguien puede ayudarme a encontrar X | O ^ | X ? Por supuesto, no espero O ^ ser un operador 'agradable', ya que no define un estado correctamente normalizado. Espero que sea algún tipo de distribución, cuyo núcleo me gustaría saber.

ACTUALIZAR 1

Algunas aclaraciones sobre el enunciado de la pregunta:

  • La pregunta es sobre un 1 -d Problema QM, con un solo grado de libertad. En principio, NO está relacionado con QFT.
  • Partiendo de la línea donde elegí la forma particular del hamiltoniano (QHO) y del estado fundamental, la definición de vacío es a ^ | 0 = 0 , dónde a ^ es el operador habitual de aniquilación de QHO.

ACTUALIZAR 2

@AccidentalFourierTransform ha sugerido lo siguiente:

X | O ^ | X = π 1 / 4 mi + ( X ) 2 / 2 mi i X ( X X ) / ( 2 π )

Siendo sustituido en la línea anterior, esto conduce claramente al resultado correcto. Ahora mi pregunta es si tal definición puede definir un operador (agradable al menos en cierto sentido). Lo que me confunde es el factor mi + ( X ) 2 / 2 junto con el hecho de que nos estamos integrando X .

¿Ha pensado en proyectar el estado propio de la posición sobre la base del número QHO?
@ alfred-centauri, esa no era mi intención original, pero intentaré ver si ayuda.
Acepto que mi condición no especifica el operador de manera única. Mi sugerencia natural es arreglarlo requiriendo O ^ = O ^ .
Buen punto. Pensando ahora en algún requisito razonable. En QFT, los estados de posición/momento definidos surgen como estados propios de los operadores correspondientes, pero en QM no tenemos una analogía de esto.

Respuestas (1)

Bueno, usted pidió un comienzo, no un buen operador, pero sospecho que alguna revisión o artículo coherente del operador de estado/desplazamiento lo tiene muy bien. Me apresuro a enmarcar el problema, y ​​usted podría optar por llevarlo a cabo de manera más eficiente.

Está buscando un operador de cambio de representación O ^ ( X ) llevándolo de los estados de Fock (estados propios del operador número/energía) a los estados propios del operador de posición X ^ con valor propio x . El operador será una función de x y operadores de creación.

Uso las convenciones de Sakurai & Napolitano, QM, (2.3.21),

| norte = a   norte norte ! | 0
y, significativamente, dado que está utilizando todo el espacio de Fock, (2.3.32),
norte | X = 1 π 1 / 4 2 norte   norte !   ( X X ) norte   mi X 2 / 2   .

Luego postulas tu cambio de base,

| X = O ^ ( X ) | 0 = 0 C norte ( X ) | norte ,
y use la relación anterior, como lo hizo para n =0 para determinar los coeficientes c(x) .

La buena noticia es que la respuesta,

| X = 0 1 π 1 / 4   ( X X ) norte a   norte 2 norte / 2 norte ! mi X 2 / 2 | 0
exponencialmente, tan a menudo,
O ^ ( X ) = 1 π 1 / 4 mi ( X X )   a / 2   mi X 2 / 2 ,
y la exponencial de los operadores es fácil de dividir mediante el uso de la identidad CBH degenerada, ya que [ X , ] = 1 , entonces Exp ( a   2 / 4 ) Exp ( X a / 2 ) Exp ( a X / 2 ) . Y por lo tanto, obtienes el exponencial del gradiente ∂ a la derecha, por lo tanto, un operador de traducción: traduce el exponente de x -Gaussian a ( X a / 2 ) 2 / 2 .

Así que finalmente,

O ^ ( X ) = 1 π 1 / 4 mi X 2 / 2 mi 2 X a mi a   2 / 2 = mi X 2 / 2 π 1 / 4 mi ( a 2 X ) 2 / 2 .
Como broma, puede comprobar que, dado que a actúa como una escritura de derivación a ,
( a + a ) 2   O ^ ( X ) | 0 = X   O ^ ( X ) | 0   ,
entonces un estado propio de X ^ , en efecto.

No he comprobado la delta-ortogonalidad de y | X estados aquí.

Editar : descubrí hoy que esto no es más que Prob. 14.4.a) del libro de M Schwartz sobre QFT. En cualquier caso, calculando < x | p > en 312004 produce la onda plana, e insertando estados de momento completos e integrando sobre ellos produce la normalización de la función δ y | X d ( X y ) buscado.

Edición II : esto es, de hecho, reducible a la célebre transformación Segal-Bargmann , Def 2 y Corolario 1, en caso de que desee seguirlo de manera más formal y pegarle etiquetas.

Edición III : me preguntan repetidamente sobre la conexión de este vacío oscilador | 0 al vacío traduccionalmente invariable del libro de Dirac, el impresionante ket estándar ,   límite pag 0 | pag | ϖ   , para cual   pag ^ | ϖ = 0   y   X | ϖ = 1 / 2 π   , así como   X | X ^ | ϖ = X / 2 π .

La relación es en realidad

| ϖ = Exp ( a a / 2 ) | 0 1 ( π ) 1 / 4   ,
implícito en lo anterior, pero evidente en esta respuesta oscuramente colocada . Por inspección, este estado está en el núcleo de 2 pag ^ = a a .